$ \newcommand{\LEQ}{\leqq} \newcommand{\HIRANO}{\unicode[sans-serif,STIXGeneral]{x306E}} \newcommand{\GEQ}{\geqq} $

 2018-06-03    問題    有理数    数列    極限  

 有理数の数列……

問題

有理数の数列\(a_1,a_2,a_3,\ldots\)と有理数の数列\(b_1,b_2,b_3,\ldots\)は、 \[ a_1 \LEQ a_2 \LEQ a_3 \LEQ \cdots \LEQ b_3 \LEQ b_2 \LEQ b_1 \] を満たし、さらに、 \[ n \to \infty \quad\text{$\HIRANO$とき}\quad b_n - a_n \to 0 \] とします。この場合、 「どんな正の整数\(n\)に対しても\(a_n \LEQ q \LEQ b_n\)を満たす」 という有理数\(q\)が存在するといえますか。

解答表示

解答

いえません。

解説

有理数の数列\(a_1, a_2, a_3, \ldots\)と、 有理数の数列\(b_1, b_2, b_3, \ldots\)がどちらも無理数\(r\)に収束するとき、 「どんな正の整数\(n\)に対しても\(a_n \LEQ q \LEQ b_n\)を満たす」 という有理数\(q\)は存在しません。

たとえば、 \(a_n\)は無理数\(\sqrt{2}\)を十進法で表記したときの小数点以下第\(n\)桁目までを切り出したものとし、 \(b_n = a_n + 10^{-n}\)とします。

\[ \begin{align*} a_1 &= 1.4 & b_1 &= 1.5 \\ a_2 &= 1.41 & b_2 &= 1.42 \\ a_3 &= 1.414 & b_3 &= 1.415 \\ a_4 &= 1.4142 & b_4 &= 1.4143 \\ a_5 &= 1.41421 & b_5 &= 1.41422 \\ a_6 &= 1.414213 & b_5 &= 1.414214 \\ &\,\,\vdots & & \,\,\vdots \end{align*} \]

このとき二つの数列は与えられた条件を満たしますが、 「どんな正の整数\(n\)に対しても\(a_n \leqq q \leqq b_n\)を満たす」 という有理数\(q\)は存在しません。

注意1

「どんな正の整数\(n\)に対しても\(a_n \LEQ r \LEQ b_n\)を満たす」という実数\(r\)が存在するといえますか、という問いに対しては「いえます」が答えになります。

注意2

問題文は、 \[ \exists q\, \forall n\, \left[ a_n \LEQ q \LEQ b_n \right] \] を問うています。これを、 \[ \forall n\, \exists q\, \left[ a_n \LEQ q \LEQ b_n \right] \] と誤解しないようにしましょう。 \(q = (a_n + b_n)/2\)とすれば自明に有理数\(q\)が存在するのではないかというのは誤解です。

注意3

問題文中の不等式、 \[ a_1 \LEQ a_2 \LEQ a_3 \LEQ \cdots \LEQ b_3 \LEQ b_2 \LEQ b_1 \] で言いたかったのは、任意の正の整数\(n\)について、 \[ a_n \LEQ b_n,\quad a_n \LEQ a_{n+1},\quad b_n \GEQ b_{n+1} \] ということでした。

ツイート


 2018-06-03    問題    有理数    数列    極限